LSAT and Law School Admissions Forum

Get expert LSAT preparation and law school admissions advice from PowerScore Test Preparation.

 Administrator
PowerScore Staff
  • PowerScore Staff
  • Posts: 8916
  • Joined: Feb 02, 2011
|
#81130
Complete Question Explanation

Weaken. The correct answer choice is (A).

Answer choice (A): This is the correct answer choice.

Answer choice (B):

Answer choice (C):

Answer choice (D):

Answer choice (E):

This explanation is still in progress. Please post any questions below!
 jlam061695
  • Posts: 62
  • Joined: Sep 17, 2016
|
#30963
I am really confused as to why A is the correct answer? If I understand correctly, the task here is to weaken the premises that state that crosswalks give the reassurance to pedestrians that cars will follow signals and that these sorts of pedestrians are less likely to look both ways when crossing. How exactly does stating that the majority of pedestrians in high traffic areas (which is just a subset of all pedestrians who cross streets) cross streets in crosswalks have any effect on the rate of pedestrian-car accidents?
User avatar
 Jonathan Evans
PowerScore Staff
  • PowerScore Staff
  • Posts: 726
  • Joined: Jun 09, 2016
|
#30974
Hi, Jlam,

Actually, the task here is to undermine the conclusion that the higher number of car/pedestrian collisions "is because of" pedestrians' false sense of security when crossing in crosswalks.

Do you notice the kind of language used here? "Because of" indicates that we are dealing with causal reasoning! Remember to think of all the possible problems with causal situations. Was it reverse causation? Does the effect occur without the purported cause? Does the cause occur without the purported effect? Might there be an alternate cause for the proposed phenomenon?

In this case, we encounter the last of these flaws. In other words, what if there was another reason why there are more car/pedestrian collisions in crosswalks than in the middle of the street?

The scenario outlined in Answer Choice (A) provides an alternative. What if the reason why there are more collisions in crosswalks is simply because there are far more pedestrians who cross in crosswalks than pedestrians who jaywalk. There need not be an increased likelihood of a collision in a crosswalk. Instead, the sheer number of pedestrians in crosswalks means that more collisions are bound to happen, even if such collisions occur a lower percentage of the time than they do when pedestrians jaywalk.

You will notice here that this question also brings up the distinction between numbers and percentages, how comparisons involving numbers and percentages can also introduce logical flaws. In addition to reviewing causal flaws, you should also review the use of numbers and percentages in LR problems. I hope this helps!
 Rita
  • Posts: 38
  • Joined: Sep 30, 2016
|
#31105
Hi Jonathan,

Could you please explain why answer choice C is incorrect? I thought it weakened the conclusion by showing that the problem is not pedestrian's overly strong sense of security that cars will follow the signals, but rather that the signals malfunction more than is to be expected, and therefore cars following the misleading signals hit pedestrians.

Thanks!
 Adam Tyson
PowerScore Staff
  • PowerScore Staff
  • Posts: 5153
  • Joined: Apr 14, 2011
|
#31130
I think you're on to something there, Rita, and if we didn't have the better answer in A, C might be a contender. Traffic signals malfunctioning at a higher than expected rate might be the alternate cause for collisions in crosswalks instead of pedestrians not paying attention - but why does that happen more often in crosswalks than out of them? Answer C really doesn't help explain that discrepancy. Why would malfunctioning signals lead to more collisions than in places where there are no signals at all, unless it's due to the differing levels of care exercised by the pedestrians? We end up pretty much right back where we started, not having strengthened anything.

This one is all about numbers. Let's say 10 people are hit in the crosswalk, and 5 are hit outside the crosswalk. Out of how many? Answer A suggests that the first group is much larger than the second - let's say those 10 are out of 10,000, and those 5 are out of 10. The larger number of folks being hit in the crosswalk may simply be due to the much larger opportunity for that to happen! No need to blame careless pedestrians putting too much trust in the signals, it's just something that happens sometimes.
 asalmen
  • Posts: 6
  • Joined: Jun 04, 2017
|
#39241
Could someone please explain why D is incorrect here? Wouldn't more drivers being alert around crosswalks provide an alternative to the author's conclusion, thus undermining it? I understand that answer choice A gives an alternative as well, just by stating that far more people cross at cross walks, and that's why the numbers of people struck are higher there but answer choice D just seems to destroy the entire argument. Am I confused because answer choice D attacks the validity of the study more so than the author's argument?
 Francis O'Rourke
PowerScore Staff
  • PowerScore Staff
  • Posts: 471
  • Joined: Mar 10, 2017
|
#39422
Hi Asalmen,

Choice (D) strenghten's the speaker's argument that the pedestrians' sense of security is the cause of the accidents. This answer choice tells us that drivers are generally alert when they are near crosswalks, so they it must not be driver inattentiveness that is at fault.

We are not looking to attack the validity of the study. The speaker's conclusion is an explanation of the study, so we want to attack this explanation.
 jayzbrisk
  • Posts: 12
  • Joined: Mar 19, 2019
|
#65597
Isn't Answer choice A a statement about high traffic areas and not pedestrians? In other words, just because this happens in high traffic areas doesn't mean we know anything about non-high traffic areas. (classic idea of formal logic) If so, then maybe in the non-high traffic areas there are many more people crossing or are hit outside of crosswalks which would make up for all the people crossing in crosswalks in high traffic areas.

Let me give some numbers to help explain what I'm saying:

Say there are 100 pedestrians who cross in high traffic crosswalks and 10 get hit, while out of the 50 who dont use those crosswalks only 5 are hit then you might say that although there more who were hit in the crosswalk the percentage still remains the same and therefore the crosswalks aren't the cause.

However if we also take into account what happens in non-high traffic areas that may not be true:

Theoretically, if in non-high traffic areas there are 10 out of 100 that are hit in the crosswalks while only 1 out of 150 pedestrians who didn't use the crosswalks are hit, then the fact that the percentage is equal in high traffic areas wouldn't undermine the argument that the crosswalks were the cause. In total, we have the following numbers:

Crosswalks: 20/200

Outside of crosswalks: 6/200

The only way I see answer A as a contender is to assume that it is more likely to get hit in a high traffic area but I think that is too much of an outside assumption for a weakener. Hence i chose choice C because that directly undermines the argument itself by giving another possible cause.

Please explain why I am wrong. :hmm:

Thank you
 Adam Tyson
PowerScore Staff
  • PowerScore Staff
  • Posts: 5153
  • Joined: Apr 14, 2011
|
#65662
Hey there jayzbrisk, that was a lot of analysis, but let me get to something you said at the very end:
The only way I see answer A as a contender is to assume that it is more likely to get hit in a high traffic area
Is that really too much of an outside assumption? If I asked you to cross a street with high traffic, like for example a busy downtown street in a city during rush hour, wouldn't you expect a greater sense of danger than if you were to cross, say, a one-lane farm road in the middle of nowhere?

Answer A does require the assumption that the danger of being hit by a car is greater when there are more cars than when there are relatively few, but that is not a big outside assumption. Rather, that sounds a lot more like common sense, the kind of outside info that you are expected to bring to bear on the LSAT, like knowing that animals require food and water and air to survive, or that if the sun is shining then it is probably daytime.

While we could certainly come up with numbers that suggest that answer A does not disprove the conclusion here, that is not the standard to apply to a weaken question. Instead, all we need is some element of doubt - in this case, the possibility that there is some other cause for the results in the stimulus. Answer A suggests a possible alternate cause for the numeric info given, and that is enough for it to weaken the argument, even if it is not completely disproven. If the cause for the disparity could be just that more people cross in crosswalks in areas where the danger is at its highest, then we cannot be so sure that "overconfident/not paying attention" is the cause.

Remember, to weaken, don't try to disprove! Just look for the answer that introduces an element of doubt.
 jayzbrisk
  • Posts: 12
  • Joined: Mar 19, 2019
|
#65676
Adam Tyson wrote:Hey there jayzbrisk, that was a lot of analysis, but let me get to something you said at the very end:
The only way I see answer A as a contender is to assume that it is more likely to get hit in a high traffic area
Is that really too much of an outside assumption? If I asked you to cross a street with high traffic, like for example a busy downtown street in a city during rush hour, wouldn't you expect a greater sense of danger than if you were to cross, say, a one-lane farm road in the middle of nowhere?
I have to admit that I wasn't actually sure if this assumption was true. I had thought that one can just as easily assume that in high traffic areas the cars move slower or maybe drivers are more careful and therefore there may be a lower risk of accidents....therefore, not knowing which way to go, I didn't pick this answer. I have been taught that if an answer can viewed as either a strengthener or a weakener depending on your own analysis its not going to be the answer to the strengthener/weaker question. Am I wrong about this approach or am I incorrectly applying it to this question?

Thanks for all your help.

Get the most out of your LSAT Prep Plus subscription.

Analyze and track your performance with our Testing and Analytics Package.